数酸

数学に関して書き留めておこうと思ったものを気まぐれに。

負の進法による整数表現

 

{M}を2以上の整数として以下のような{-M}進法による整数の表示が考えられる。

{\displaystyle (a_n a_{n-1}...a_0)_{-M}=\sum_{k=0}^n a_k(-M)^k }  

但し { a_k \in \{ 0,1,2,..,M-1 \} } 

{0}以外の表示においては {a_n \neq 0} 

命題 {-M}進法によって任意の整数が一意に表現できる

 

ことを確認する。

 証明

まず一意性を証明する。移項することで

{\displaystyle \sum_{k=0}^n b_k(-M)^k }=0  

{ b_k \in \{-(M-1),..,-1,0,1,..,M-1 \} } 

ならば任意の{k}{b_k=0}を示せばよい。

そうでないと仮定すると{b_k \neq 0}となる最小の{k}が存在するので、それを{k_0}とする。{(-M)^{k_0} }で割ると

{\displaystyle b_{k_0}-M \sum_{k=k_0+1}^n b_k(-M)^{k-k_0-1 }=0} 

左辺は明らかに{M}の倍数でないので矛盾する。


次に任意の整数が表現できることを示す。

整数{A}に対して{A-a}{M}の倍数となるような{a \in \{ 0,1,..,M-1 \}}が存在する。もし{B=(A-a)/(-M)}{-M}進法表示可能ならば{A}{-M}進法表示可能。実際

{\displaystyle B=(a_n a_{n-1}...a_0)_{-M} }

なら

{\displaystyle A=(a_n a_{n-1}...a_0 a)_{-M} }

と表示できる。

{\displaystyle |B| \lt |A| } 

{\displaystyle \Leftrightarrow |A-a| \lt M|A| }

{\displaystyle \Leftarrow |A|+a \lt M|A| }

{\displaystyle \Leftrightarrow \frac{a}{M-1} \lt |A| }

であるから{|A| \gt 1}ならば{A}の表示可能性は{A}より絶対値の小さい整数の表示可能性に帰着する。{0,1}は明らかなので{-1}{-M}進法表示できることだけ確認すればよいが

{\displaystyle -1=(M-1)+1 \times (-M)=(1,M-1)_{-M} }

となる。□

指数分布の最大値分布

soratobipenguin.hatenablog.com

リンク先に書き直しました。こちらも一応残しておきますが、しなくてよい面倒な計算をしています。

 

{X_1,X_2,..,X_n \sim {\rm Exp}(\lambda)} 互いに独立

{Y_n=\max (X_1,X_2,..,X_n)}

このとき{Y_n} の分布について考えてみる。

 

期待値を求めてみよう。

{\displaystyle E[Y_n]=\lambda^{-1} \sum_{k=1}^n \frac{1}{k} }

 

ちなみに最小値分布{\min (X_1,X_2,..,Xn)}{ {\rm Exp}(n \lambda )}に従うことが簡単に計算できるので期待値は{(n \lambda)^{-1}} となる。

{Y_n}の分布関数を{F_n}とすると

{ \displaystyle P(\max(X_1,X_2,..,X_n) \leq x )}

{ \displaystyle =P(X_1 \leq x)P(X_2 \leq x)...P(X_n \leq x)}

{ \displaystyle =P(X_1 \leq x)^n }

であるから{x \geq 0}において

{\displaystyle F_n(x)=(1-e^{- \lambda x})^n}

その他のの範囲では0。

したがって

{\displaystyle E[Y_n ]=\int_0^{+\infty} (1-F_n(x))dx } 

{\displaystyle =\int_0^{+\infty} \sum_{k=1}^n  {}_n C_k (-1)^{k-1}  e^{-k \lambda x}dx }

{\displaystyle =\lambda^{-1}\sum_{k=1}^n \frac{{}_n C_k (-1)^{k-1}}{k} }


 一方で

{\displaystyle \sum_{k=1}^n \frac{1}{k} }

{\displaystyle =\int_0^1 (1+t+t^2+...+t^{n-1})dt }

{\displaystyle =\int_0^1 \frac{1-t^n}{1-t}dt }

{t=1-x} と置いて

{\displaystyle =\int_0^1 \frac{1-(1-x)^n}{x} dx}

{\displaystyle =\int_0^1 \sum_{k=1}^n {}_n C_k  (-x)^{k-1} dx }

{\displaystyle =\sum_{k=1}^n \frac{ {}_n C_k (-1)^{k-1} }{k} }

と計算される。□

 

次に分散を求める。

{\displaystyle V(Y_n)=\lambda^{-2} \sum_{k=1}^n \frac{1}{k^2} }

 

 この証明には

に登場する以下の式を用いる。

{\displaystyle  \sum_{k=1}^n \frac{(-1)^{k-1} {}_nC_k }{k^m}= \sum_{p_1+...+p_n=m} \frac{1}{1^{p_1}2^{p_2}...n^{p_n}}  }

但し{ p_1,p_2,...,p_n }は非負整数を動く。 

 

期待値の場合と同様にして

{\displaystyle E[Y_n^2]=\int_0^{+\infty} 2x(1-F_n(x))dx } 

{\displaystyle =\int_0^{+\infty} \sum_{k=1}^n  {}_n C_k (-1)^{k-1}  2xe^{-k \lambda x}dx }

{\displaystyle =2\lambda^{-2}\sum_{k=1}^n \frac{{}_n C_k (-1)^{k-1}}{k^2} }

 


一方、引用した式に{ m=2 }を代入して

{\displaystyle  \sum_{k=1}^n \frac{(-1)^{k-1} {}_nC_k }{k^2} }

{\displaystyle  =\sum_{p_1+...+p_n=2} \frac{1}{1^{p_1}2^{p_2}...n^{p_n}}  }

{\displaystyle =\frac{1}{2}((\sum_{k=1}^n k^{-1})^2+\sum_{k=1}^n k^{-2}) }

 


よって

 {\displaystyle V(Y_n)=E[Y_n^2]-E[Y_n]^2 }

{\displaystyle =\lambda^{-2}((\sum_{k=1}^n k^{-1})^2+\sum_{k=1}^n k^{-2})-(\lambda^{-1} \sum_{k=1}^n k^{-1})^2  }

{\displaystyle =\lambda^{-2} \sum_{k=1}^n k^{-2} }

 

同じようにして高次のモーメントも計算可能。3次、4次の結果を記しておくと{  \mu=E[Y_n]}として

{\displaystyle E[ (Y_n-\mu)^3 ]=2\lambda^{-3} \sum_{k=1}^n k^{-3} }

{\displaystyle E[ (Y_n-\mu)^4 ]=6\lambda^{-4} \sum_{k=1}^n k^{-4}+3\lambda^{-4} (\sum_{k=1}^n k^{-2})^2 }

 

 今度は{n}が大きくなる時に近づいていく分布を求める。

{\displaystyle Y_n-\lambda^{-1} \log n \rightarrow ^d Z  (n \rightarrow \infty) }

但し{Z}の分布関数{G}

{\displaystyle G(x)= \exp(-e^{-\lambda x  })} 

 

 {Z_n=Y_n-\lambda^{-1} \log n}の分布関数を{G_n}とする。

{F_n(x)=(1-e^{- \lambda x})^n \chi_ { [ 0,\infty  )}(x) }

{\chi}は定義関数。

{G_n(x)=F_n(x+\lambda^{-1} \log n)}

{=(1-\frac{e^{-\lambda x}}{n})^n \chi_{ [-\lambda^{-1} \log n,\infty)}(x) }

これは { n \rightarrow \infty } によって任意の実数{x}

{G(x)= \exp(-e^{-\lambda x})  }  に収束する。□

 

 ついでに{E[Z],V(Z)}を計算しておくと

{E[ Z_n ] \rightarrow E[Z] } が比較的容易に分かるので

{\displaystyle E[Z]=\lim_{n \rightarrow \infty }(E [ Y_n ] -\lambda^{-1} \log n) }

{\displaystyle =\lambda^{-1} \lim_{n \rightarrow \infty } (\sum_{k=1}^n \frac{1}{k}-\log n)}

{\displaystyle=\gamma} \lambda^{-1}

となる({ \gamma}オイラー定数)。

同じようにして

{\displaystyle V(Z)=\lambda^{-2} \sum_{k=1}^\infty \frac{1}{k^2}= \frac{\pi^2}{6}\lambda^{-2} }

ついでに

{\displaystyle E[ (Z-\gamma \lambda^{-1})^3 ]=2 \zeta(3) \lambda^{-3}}

{\displaystyle E[ (Z-\gamma \lambda^{-1})^4 ]=6\lambda^{-4} \zeta(4)+3\lambda^{-4} \zeta(2)^2=\frac{3\pi^4}{20} \lambda^{-4}}